LSAT and Law School Admissions Forum

Get expert LSAT preparation and law school admissions advice from PowerScore Test Preparation.

User avatar
 Dave Killoran
PowerScore Staff
  • PowerScore Staff
  • Posts: 5852
  • Joined: Mar 25, 2011
|
#80196
Complete Question Explanation
(The complete setup for this game can be found here: lsat/viewtopic.php?t=885)

The correct answer choice is (B)

As discussed during the setup, due to the combination of the third rule and the last rule, light 3 must always be on. Consequently, answer choice (B) is correct.
 EsquireGirl
  • Posts: 14
  • Joined: Jun 27, 2017
|
#46035
Hi,

Why must light number 3 be on? It says that "two of the lights on the north side are on." That implies: 1) either light 1 or light 3 is on and 2) light 2 is always on. Doesn't it?

Thanks,
User avatar
 Dave Killoran
PowerScore Staff
  • PowerScore Staff
  • Posts: 5852
  • Joined: Mar 25, 2011
|
#46060
Hi Esquire,

Thanks for the question! This is addressed in our setup discussion to the game, over here: lsat/viewtopic.php?t=885

As discussed there, there is a key inference in the game. The final rule states that two lights on the north side are on. From the third rule we know that lights 1 and 2 cannot be on at the same time, so, by Hurdling the Uncertainty we can infer that light 3 must always be on (otherwise you could not fulfill the constraints of this rule). This inference answers question #7 and helps to answer several others.

Thanks!

Get the most out of your LSAT Prep Plus subscription.

Analyze and track your performance with our Testing and Analytics Package.